Difference between revisions of "2013 AIME I Problems/Problem 5"
m (→Solution 4) |
(→Solutions) |
||
Line 4: | Line 4: | ||
== Solutions == | == Solutions == | ||
=== Solution 1 === | === Solution 1 === | ||
− | We have that <math>9x^3 = (x+1)^3</math>, so it follows that <math>\sqrt[3]{9}x = x+1</math>. Solving for <math>x</math> yields <math>\frac{1}{\sqrt[3]{9}-1} = \frac{\sqrt[3]{81}+\sqrt[3]{9}+1}{8}</math>, so the answer is <math>\boxed{ | + | We have that <math>9x^3 = (x+1)^3</math>, so it follows that <math>\sqrt[3]{9}x = x+1</math>. Solving for <math>x</math> yields <math>\frac{1}{\sqrt[3]{9}-1} = \frac{\sqrt[3]{81}+\sqrt[3]{9}+1}{8}</math>, so the answer is <math>\boxed{098}</math>. |
=== Solution 2 === | === Solution 2 === | ||
− | Let <math>r</math> be the real root of the given [[polynomial]]. Now define the cubic polynomial <math>Q(x)=-x^3-3x^2-3x+8</math>. Note that <math>1/r</math> must be a root of <math>Q</math>. However we can simplify <math>Q</math> as <math>Q(x)=9-(x+1)^3</math>, so we must have that <math>(\frac{1}{r}+1)^3=9</math>. Thus <math>\frac{1}{r}=\sqrt[3]{9}-1</math>, and <math>r=\frac{1}{\sqrt[3]{9}-1}</math>. We can then multiply the numerator and denominator of <math>r</math> by <math>\sqrt[3]{81}+\sqrt[3]{9}+1</math> to rationalize the denominator, and we therefore have <math>r=\frac{\sqrt[3]{81}+\sqrt[3]{9}+1}{8}</math>, and the answer is <math>\boxed{ | + | Let <math>r</math> be the real root of the given [[polynomial]]. Now define the cubic polynomial <math>Q(x)=-x^3-3x^2-3x+8</math>. Note that <math>1/r</math> must be a root of <math>Q</math>. However we can simplify <math>Q</math> as <math>Q(x)=9-(x+1)^3</math>, so we must have that <math>(\frac{1}{r}+1)^3=9</math>. Thus <math>\frac{1}{r}=\sqrt[3]{9}-1</math>, and <math>r=\frac{1}{\sqrt[3]{9}-1}</math>. We can then multiply the numerator and denominator of <math>r</math> by <math>\sqrt[3]{81}+\sqrt[3]{9}+1</math> to rationalize the denominator, and we therefore have <math>r=\frac{\sqrt[3]{81}+\sqrt[3]{9}+1}{8}</math>, and the answer is <math>\boxed{098}</math>. |
=== Solution 3 === | === Solution 3 === | ||
Line 13: | Line 13: | ||
Letting <math>r</math> be the root, we clearly we have <math>r + f_1(r) + f_2(r) = \frac{3}{8}</math> by Vieta's. Thus it follows <math>c=8</math>. | Letting <math>r</math> be the root, we clearly we have <math>r + f_1(r) + f_2(r) = \frac{3}{8}</math> by Vieta's. Thus it follows <math>c=8</math>. | ||
− | Now, note that <math>\sqrt[3]{a} + \sqrt[3]{b} + 1</math> is a root of <math>x^3 - 3x^2 - 24x - 64 = 0</math>. Thus <math>(x-1)^3 = 27x + 63</math> so <math>(\sqrt[3]{a} + \sqrt[3]{b})^3 = 27(\sqrt[3]{a} + \sqrt[3]{b}) + 90</math>. Checking the non-cubicroot dimension part, we get <math>a + b = 90</math> so it follows that <math>a + b + c = \boxed{ | + | Now, note that <math>\sqrt[3]{a} + \sqrt[3]{b} + 1</math> is a root of <math>x^3 - 3x^2 - 24x - 64 = 0</math>. Thus <math>(x-1)^3 = 27x + 63</math> so <math>(\sqrt[3]{a} + \sqrt[3]{b})^3 = 27(\sqrt[3]{a} + \sqrt[3]{b}) + 90</math>. Checking the non-cubicroot dimension part, we get <math>a + b = 90</math> so it follows that <math>a + b + c = \boxed{098}</math>. |
=== Solution 4 === | === Solution 4 === | ||
Line 29: | Line 29: | ||
<cmath>=\sqrt[3]{\dfrac{45}{512}+\dfrac{36}{512}}+\sqrt[3]{\dfrac{45}{512}-\dfrac{36}{512}}+\dfrac{1}{8}</cmath> | <cmath>=\sqrt[3]{\dfrac{45}{512}+\dfrac{36}{512}}+\sqrt[3]{\dfrac{45}{512}-\dfrac{36}{512}}+\dfrac{1}{8}</cmath> | ||
<cmath>=\dfrac{\sqrt[3]{81}}{8}+\dfrac{\sqrt[3]{9}}{8}+\dfrac{1}{8}=\dfrac{\sqrt[3]{81}+\sqrt[3]{9}+1}{8}</cmath> | <cmath>=\dfrac{\sqrt[3]{81}}{8}+\dfrac{\sqrt[3]{9}}{8}+\dfrac{1}{8}=\dfrac{\sqrt[3]{81}+\sqrt[3]{9}+1}{8}</cmath> | ||
− | and hence the answer is <math>81+9+8=\boxed{ | + | and hence the answer is <math>81+9+8=\boxed{098}</math>. |
== See Also == | == See Also == |
Revision as of 11:41, 17 March 2013
Problem
The real root of the equation can be written in the form , where , , and are positive integers. Find .
Contents
Solutions
Solution 1
We have that , so it follows that . Solving for yields , so the answer is .
Solution 2
Let be the real root of the given polynomial. Now define the cubic polynomial . Note that must be a root of . However we can simplify as , so we must have that . Thus , and . We can then multiply the numerator and denominator of by to rationalize the denominator, and we therefore have , and the answer is .
Solution 3
It is clear that for the algebraic degree of to be that there exists some cubefree integer and positive integers such that and (it is possible that , but then the problem wouldn't ask for both an and ). Let be the automorpism over which sends and which sends (note : is a cubic root of unity).
Letting be the root, we clearly we have by Vieta's. Thus it follows . Now, note that is a root of . Thus so . Checking the non-cubicroot dimension part, we get so it follows that .
Solution 4
We proceed by using the cubic formula.
Let , , , and . Then let and . Then the real root of is Now note that and Thus and hence the answer is .
See Also
2013 AIME I (Problems • Answer Key • Resources) | ||
Preceded by Problem 4 |
Followed by Problem 6 | |
1 • 2 • 3 • 4 • 5 • 6 • 7 • 8 • 9 • 10 • 11 • 12 • 13 • 14 • 15 | ||
All AIME Problems and Solutions |